LSAT and Law School Admissions Forum

Get expert LSAT preparation and law school admissions advice from PowerScore Test Preparation.

 eober
  • Posts: 107
  • Joined: Jul 24, 2014
|
#16556
Hi,

I was wondering how answer choice B fails to solve the paradox. If there are antitheft equipments now, then there would be less cost (since stolen books would cost stores) to the businesses. And also how does A solves this paradox (decrease in readers and an increase in profits)

Thanks!
 Robert Carroll
PowerScore Staff
  • PowerScore Staff
  • Posts: 1787
  • Joined: Dec 06, 2013
|
#16568
eober,

Answer choice (B) fails to resolve the paradox because it doesn't explain why bookstores are making MORE money now. The antitheft devices have helped bookstores remain largely unaffected by an increase in shoplifting, but this doesn't explain why their profits went up, just why they wouldn't be affected as badly by shoplifting as are other retail outlets. There is no reason to think that bookstores' being unaffected by this would increase profits - at best, it would just be expected to keep them at the same position they were before the recent increase in shoplifting. Given that the average number of books read has declined, it's still paradoxical that bookstores are actually doing better now. It's not as if they stopped shoplifting that had been going on; remember, the shoplifting increase is also recent!

Answer choice (A) resolves the paradox because library books provide a way for people to read books without actually buying them themselves - one purchase can provide a large number of people with the ability to read a book. If cuts in government spending have reduced the ability of libraries to keep up with new books, it makes sense that people would now have to buy those books because they can't easily get them at the library anymore. So even if people are reading less, they also can't depend on the library as much anymore, so bookstores are profiting!

Robert Carroll
 Tomars
  • Posts: 15
  • Joined: Aug 03, 2017
|
#37934
Hello Powerscore staff!

I understand how B doesn't resolve the paradox, but in looking at C, D, and E, it seems they only address one part of the paradox. These 3 address the increase in profit, but do not imply anything about decrease in readership. Why is this okay here?

Thanks!
 AthenaDalton
PowerScore Staff
  • PowerScore Staff
  • Posts: 296
  • Joined: May 02, 2017
|
#37961
Hi Tomars,

We're looking for answer choices that "help resolve" the paradoxical results. So it's ok if they help just a little bit -- they don't need to each completely explain the paradox. It's possible that the paradox has multiple explanations -- A, C, D, and E.

The last three answer choices explain why bookstore profits might be increasing even though the number of books that an average person reads per year has declined. Essentially we're asking, how can Barnes & Noble make more money when people are reading fewer books? We don't need an explanation for why people are reading fewer books, we just need to explain why profits are increasing.

The answer is that the bookstores are increasing profits by doing something other than just selling a greater quantity of books: (C) bookstores are making profits off of selling coffee and not just books (D) people are spending more per book and (E) people are buying magazines from bookstores rather than having them mailed to their homes.

I hope this clears things up. Good luck studying!
User avatar
 PresidentLSAT
  • Posts: 87
  • Joined: Apr 19, 2021
|
#86546
Hi,

I thought B was wrong because it doesn't explain the decrement in readership, though it does explain the increment in profits. Less shoplifting definitely makes you more money and is the overlooked source of profit. I've worked in retail management for 10 years. When people steal less, you're able to sell more products-> profit increment. If I stock up 6 items on a shelf and a shoplifter takes all three vs Walmart stocks the same amount and sells all of them, Walmart is in raking in a profit. Shoplifting is the biggest factor affecting shrink and can significantly impact your bottom line and free cash flow. I just couldn't figure out how the answer choice accountted for how people would read less. No?

Please let me know if I missed something here. I was too confident in the question lol.
User avatar
 KelseyWoods
PowerScore Staff
  • PowerScore Staff
  • Posts: 1079
  • Joined: Jun 26, 2013
|
#86671
Hi PresidentLSAT!

Answer choice (B) doesn't actually say that bookstores experienced less shoplifting--it just says that they were largely unaffected by the increase in shoplifting that was impacting other stores. For all we know, there was zero shoplifting before the antitheft equipment was installed and so it just prevented them from being a victim to shoplifting later but did not actually increase their profits (and maybe even decreased their profits since they had to pay for the antitheft equipment). So answer choice (B) doesn't explain how bookstores are making more money even though people are reading less.

Hope this helps!

Best,
Kelsey
User avatar
 PresidentLSAT
  • Posts: 87
  • Joined: Apr 19, 2021
|
#86673
KelseyWoods wrote: Mon Apr 26, 2021 7:31 pm Hi PresidentLSAT!

Answer choice (B) doesn't actually say that bookstores experienced less shoplifting--it just says that they were largely unaffected by the increase in shoplifting that was impacting other stores. For all we know, there was zero shoplifting before the antitheft equipment was installed and so it just prevented them from being a victim to shoplifting later but did not actually increase their profits (and maybe even decreased their profits since they had to pay for the antitheft equipment). So answer choice (B) doesn't explain how bookstores are making more money even though people are reading less.

Hope this helps!

Best,
Kelsey
Thank you. It does.
User avatar
 LSAT4Life
  • Posts: 17
  • Joined: Aug 10, 2021
|
#90035
Hi, I have a more general question. For this type of question, I prephrased as the answer should explain why books read decreased but the bookstore profits increased. Do I need to go further in prephrasing?
User avatar
 evelineliu
PowerScore Staff
  • PowerScore Staff
  • Posts: 91
  • Joined: Sep 06, 2021
|
#90208
Hi LSAT4Life,

For Except questions on Logical Reasoning, I try to prephrase the four incorrect answers. Incorrect answers will each provide new information about book purchasers or bookstores that explain how bookstores profit more even though people read fewer books. Possible reasons: increased profits are due to sale of products other than books (magazines, games, beverages). Bookstores could also mark up prices even higher so they can profit more even though they sell fewer books.

The correct answer choice (B) will be irrelevant/outside the scope.

Hope that helps!
Eveline
User avatar
 Relaxo
  • Posts: 13
  • Joined: Jan 23, 2022
|
#94113
Hi,

I struggled between (A) and (B).

(B) This doesn't directly explain why profits increased. But it could that books were stolen in the beginning of the period, which hurt profits. Then the antitheft equipment was installed, and then, profits increased.
For this, however, one has to assume that the antitheft equipment was installed before the recent increase in shoplifting.

(A) This is fairly indirect. I see how this plays out: Fewer popular contemporary books in public libraries -> more people incentivized to buy the book.
But to make (A) work we need to assume that the books are bought in bookstores and not in pure online shops or other providers. Also, what if the contemporary novels are popular, but the base reading these novels is insiginficantly small? Then even if they buy it, the profits aren't really directly influenced ...

Get the most out of your LSAT Prep Plus subscription.

Analyze and track your performance with our Testing and Analytics Package.